4-8章习题解答
第4章 振动
P174.
4.1 一物体沿x轴做简谐振动,振幅A = 0.12m,周期T = 2s.当t = 0时,物体的位移x = 0.06m,且向x轴正向运动.求:
(1)此简谐振动的表达式;
(2)t = T/4时物体的位置、速度和加速度;
(3)物体从x = -0.06m,向x轴负方向运动第一次回到平衡位置所需的时间.
[解答](1)设物体的简谐振动方程为
x = Acos(ωt + φ),
其中A = 0.12m,角频率ω = 2π/T = π.
当t = 0时,x = 0.06m,所以
cosφ = 0.5,
因此
φ = ±π/3.
物体的速度为
v = dx/dt = -ωAsin(ωt + φ).
当t = 0时,
v = -ωAsinφ,
由于v > 0,所以sinφ < 0,因此
φ = -π/3.
简谐振动的表达式为
x = 0.12cos(πt – π/3).
(2)当t = T/4时物体的位置为
x = 0.12cos(π/2 – π/3)
= 0.12cosπ/6 = 0.104(m).
速度为
v = -πAsin(π/2 – π/3)
= -0.12πsinπ/6 = -0.188(m·s-1).
加速度为
a = dv/dt = -ω2Acos(ωt + φ)
= -π2Acos(πt - π/3)
= -0.12π2cosπ/6 = -1.03(m·s-2).
(3)方法一:求时间差.当x = -0.06m时,可得
cos(πt1 - π/3) = -0.5,
因此
πt1 - π/3 = ±2π/3.
由于物体向x轴负方向运动,即v < 0,所以sin(πt1 - π/3) > 0,因此
πt1 - π/3 = 2π/3,
得t1 = 1s.
当物体从x = -0.06m处第一次回到平衡位置时,x = 0,v > 0,因此
cos(πt2 - π/3) = 0,
可得 πt2 - π/3 = -π/2或3π/2等.
由于t2 > 0,所以
πt2 - π/3 = 3π/2,
可得 t2 = 11/6 = 1.83(s).
所需要的时间为
Δt = t2 - t1 = 0.83(s).
方法二:反向运动.物体从x = -0.06m,向x轴负方向运动第一次回到平衡位置所需的时间就是它从x = 0.06m,即从起点向x轴正方向运动第一次回到平衡位置所需的时间.在平衡位置时,x = 0,v < 0,因此
cos(πt - π/3) = 0,
可得 πt - π/3 = π/2,
解得 t = 5/6 = 0.83(s).
[注意]根据振动方程
x = Acos(ωt + φ),
当t = 0时,可得
φ = ±arccos(x0/A),(-π < φ≦π),
初位相的取值由速度决定.
由于
v = dx/dt = -ωAsin(ωt + φ),
当t = 0时,
v = -ωAsinφ,
当v > 0时,sinφ < 0,因此
φ = -arccos(x0/A);
当v < 0时,sinφ > 0,因此
φ = arccos(x0/A).
可见:当速度大于零时,初位相取负值;当速度小于零时,初位相取正值.如果速度等于零,当初位置x0 = A时,φ = 0;当初位置x0 = -A时,φ = π.
4.2 已知一简谐振子的振动曲线如图所示,试由图求:
(1)a,b,c,d,e各点的位相,及到达这些状态的时刻t各是多少?已知周期为T;
(2)振动表达式;
(3)画出旋转矢量图.
[解答]方法一:由位相求时间.
(1)设曲线方程为
x = AcosΦ,
其中A表示振幅,Φ = ωt + φ表示相位.
由于xa = A,所以
cosΦa = 1,
因此 Φa = 0.
由于xb = A/2,所以
cosΦb = 0.5,
因此 Φb = ±π/3;
由于位相Φ随时间t增加,b点位相就应该大于a点的位相,因此
Φb = π/3.
由于xc = 0,所以
cosΦc = 0,
又由于c点位相大于b位相,因此
Φc = π/2.
同理可得其他两点位相为
Φd = 2π/3,Φe = π.
c点和a点的相位之差为π/2,时间之差为T/4,而b点和a点的相位之差为π/3,时间之差应该为T/6.因为b点的位移值与O时刻的位移值相同,所以到达a点的时刻为
ta = T/6.
到达b点的时刻为
tb = 2ta = T/3.
到达c点的时刻为
tc = ta + T/4 = 5T/12.
到达d点的时刻为
td = tc + T/12 = T/2.
到达e点的时刻为
te = ta + T/2 = 2T/3.
(2)设振动表达式为
x = Acos(ωt + φ),
当t = 0时,x = A/2时,所以
cosφ = 0.5,
因此
φ = ±π/3;
由于零时刻的位相小于a点的位相,所以
φ = -π/3,
因此振动表达式为
.
另外,在O时刻的曲线上作一切线,由于速度是位置对时间的变化率,所以切线代表速度的方向;由于其斜率大于零,所以速度大于零,因此初位相取负值,从而可得运动方程.
(3)如图旋转矢量图所示.
方法二:由时间求位相.将曲线反方向延长与t轴相交于f点,由于xf = 0,根据运动方程,可得

所以
.
显然f点的速度大于零,所以取负值,解得
tf = -T/12.
从f点到达a点经过的时间为T/4,所以到达a点的时刻为
ta = T/4 + tf = T/6,
其位相为
.
由图可以确定其他点的时刻,同理可得各点的位相.
4.3如图所示,质量为10g的子弹以速度v = 103m·s-1水平射入木块,并陷入木块中,使弹簧压缩而作简谐振动.设弹簧的倔强系数k = 8×103N·m-1,木块的质量为4.99kg,不计桌面摩擦,试求:
(1)振动的振幅;
(2)振动方程.
[解答](1)子弹射入木块时,由于时间很短,木块还来不及运动,弹簧没有被压缩,它们的动量守恒,即
mv = (m + M)v0.
解得子弹射入后的速度为
v0 = mv/(m + M) = 2(m·s-1),
这也是它们振动的初速度.
子弹和木块压缩弹簧的过程机械能守恒,可得
(m + M) v02/2 = kA2/2,
所以振幅为
= 5×10-2(m).
(2)振动的圆频率为
= 40(rad·s-1).
取木块静止的位置为原点、向右的方向为位移x的正方向,振动方程可设为
x = Acos(ωt + φ).
当t = 0时,x = 0,可得
φ = ±π/2;
由于速度为正,所以取负的初位相,因此振动方程为
x = 5×10-2cos(40t - π/2)(m).
4.4 如图所示,在倔强系数为k的弹簧下,挂一质量为M的托盘.质量为m的物体由距盘底高h处自由下落与盘发生完全非弹性碰撞,而使其作简谐振动,设两物体碰后瞬时为t = 0时刻,求振动方程.
[解答]物体落下后、碰撞前的速度为
,
物体与托盘做完全非弹簧碰撞后,根据动量守恒定律可得它们的共同速度为
,
这也是它们振动的初速度.
设振动方程为
x = Acos(ωt + φ),
其中圆频率为
.
物体没有落下之前,托盘平衡时弹簧伸长为x1,则
x1 = Mg/k.
物体与托盘碰撞之后,在新的平衡位置,弹簧伸长为x2,则
x2 = (M + m)g/k.
取新的平衡位置为原点,取向下的方向为正,则它们振动的初位移为
x0 = x1 - x2 = -mg/k.
因此振幅为

;
初位相为
.
4.5重量为P的物体用两根弹簧竖直悬挂,如图所示,各弹簧的倔强系数标明在图上.试求在图示两种情况下,系统沿竖直方向振动的固有频率.
[解答](1)可以证明:当两根弹簧串联时,总倔强系数为k = k1k2/(k1 + k2),因此固有频率为
 
.
(2)因为当两根弹簧并联时,总倔强系数等于两个弹簧的倔强系数之和,因此固有频率为
.
4.6 一匀质细圆环质量为m,半径为R,绕通过环上一点而与环平面垂直的水平光滑轴在铅垂面内作小幅度摆动,求摆动的周期.
[解答]方法一:用转动定理.通过质心垂直环面有一个轴,环绕此轴的转动惯量为
Ic = mR2.
根据平行轴定理,环绕过O点的平行轴的转动惯量为
I = Ic + mR2 = 2mR2.
当环偏离平衡位置时,重力的力矩为
M = -mgRsinθ,
方向与角度θ增加的方向相反.
根据转动定理得
Iβ = M,
即 ,
由于环做小幅度摆动,所以sinθ≈θ,可得微分方程
.
摆动的圆频率为
,
周期为
.
方法二:用机械能守恒定律.取环的质心在最底点为重力势能零点,当环心转过角度θ时,重力势能为
Ep = mg(R - Rcosθ),
绕O点的转动动能为
,
总机械能为
.
环在转动时机械能守恒,即E为常量,将上式对时间求导,利用ω = dθ/dt,β = dω/dt,得
0 = Iωβ + mgR(sinθ) ω,
由于ω ≠ 0,当θ很小有sinθ≈θ,可得振动的微分方程
,
从而可求角频率和周期.
[注意]角速度和圆频率使用同一字母ω,不要将两者混淆.
4.7 横截面均匀的光滑的U型管中有适量液体如图所示,液体的总长度为L,求液面上下微小起伏的自由振动的频率。
解:建立竖直坐标如图,令微小振动中,两臂水银面相平时,水银面坐标为0,水银的重力势能为0,则以右臂水银面的坐标为准,在振动中任一时刻,水银的运动速度.这时振动中水银的动能为,水银的势能(看作两水银面相平的状态下,从右臂移高度为y的一段水银柱到左臂,则有质量为S?y的水银升高了高度y)为S?gy2.因振动中机械能守恒
常量对t求导数可得 
化简 
这就是简谐振动的微分方程,
由此可得振动角频率 

4.8 质量为10×10-3kg的小球与轻弹簧组成的系统,按的规律作振动,式中t以秒(s)计,x以米(m)计.求:
(1)振动的圆频率、周期、振幅、初位相;
(2)振动的速度、加速度的最大值;
(3)最大回复力、振动能量、平均动能和平均势能;
(4)画出这振动的旋转矢量图,并在图上指明t为1,2,10s等各时刻的矢量位置.
[解答](1)比较简谐振动的标准方程
x = Acos(ωt + φ),
可知:圆频率为
ω =8π,
周期
T = 2π/ω = 1/4 = 0.25(s),
振幅为
A = 0.1(m),
位相为
φ = 2π/3.
(2)速度的最大值为
vm = ωA = 0.8π = 2.51(m·s-1);
加速度的最大值为
am = ω2A = 6.4π2 = 63.2(m·s-2).
(3)弹簧的倔强系数为
k = mω2,
最大回复力为
f = kA = mω2A = 0.632(N);
振动能量为
E = kA2/2 = mω2A2/2 = 3.16×10-2(J),
平均动能和平均势能为
= kA2/4 = mω2A2/4 = 1.58×10-2(J).
(4)如图所示,当t为1,2,10s等时刻时,旋转矢量的位置是相同的.
4.9 一氢原子在分子中的振动可视为简谐振动.已知氢原子质量m = 1.68×10-27kg,振动频率v = 1.0×1014Hz,振幅A = 1.0×10-11m.试计算:
(1)此氢原子的最大速度;
(2)与此振动相联系的能量.
[解答](1)氢原子的圆频率为
ω = 2πv = 6.28×1014(rad·s-1),
最大速度为
vm = ωA = 6.28×103(m·s-1).
(2)氢原子的能量为
= 3.32×10-20(J).
4.10 质量为0.25kg的物体,在弹性力作用下作简谐振动,倔强系数k = 25N·m-1,如果开始振动时具有势能0.6J,和动能0.2J,求:
(1)振幅;
(2)位移多大时,动能恰等于势能?
(3)经过平衡位置时的速度.
[解答]物体的总能量为
E = Ek + Ep = 0.8(J).
(1)根据能量公式
E = kA2/2,
得振幅为
= 0.253(m).
(2)当动能等于势能时,即Ek = Ep,由于
E = Ek + Ep,
可得
E = 2Ep,
即 ,
解得
= ±0.179(m).
(3)再根据能量公式
E = mvm2/2,
得物体经过平衡位置的速度为
= ±2.53(m·s-1).
4.11 两个质点平行于同一直线并排作同频率、同振幅的简谐振动.在振动过程中,每当它们经过振幅一半的地方时相遇,而运动方向相反.求它们的位相差,并作旋转矢量图表示.
[解答]设它们的振动方程为
x = Acos(ωt + φ),
当x = A/2时,可得位相为
ωt + φ = ±π/3.
由于它们在相遇时反相,可取
Φ1 = (ωt + φ)1 = -π/3,
Φ2 = (ωt + φ)2 = π/3,
它们的相差为
ΔΦ = Φ2 – Φ1 = 2π/3,
或者
ΔΦ` = 2π –ΔΦ = 4π/3.
矢量图如图所示.
4.12 两个频率和振幅都相同的简谐振动的x-t曲线如图所示,求:
(1)两个简谐振动的位相差;
(2)两个简谐振动的合成振动的振动方程.
[解答](1)两个简谐振动的振幅为
A = 5(cm),
周期为
T = 4(s),
圆频率为
ω =2π/T = π/2,
它们的振动方程分别为
x1 = Acosωt = 5cosπt/2,
x2 = Asinωt = 5sinπt/2 = 5cos(π/2 - πt/2)
即 x2 = 5cos(πt/2 - π/2).
位相差为
Δφ = φ2 - φ1 = -π/2.
(2)由于
x = x1 + x2 = 5cosπt/2 + 5sinπt/2
= 5(cosπt/2·cosπ/4 + 5sinπt/2·sinπ/4)/sinπ/4
合振动方程为
(cm).
4.13已知两个同方向简谐振动如下:
,
.
(1)求它们的合成振动的振幅和初位相;
(2)另有一同方向简谐振动x3 = 0.07cos(10t +φ),问φ为何值时,x1 + x3的振幅为最大?φ为何值时,x2 + x3的振幅为最小?
(3)用旋转矢量图示法表示(1)和(2)两种情况下的结果.x以米计,t以秒计.
[解答](1)根据公式,合振动的振幅为

= 8.92×10-2(m).
初位相为
= 68.22°.
(2)要使x1 + x3的振幅最大,则
cos(φ – φ1) = 1,
因此
φ – φ1 = 0,
所以
φ = φ1 = 0.6π.
要使x2 + x3的振幅最小,则
cos(φ – φ2) = -1,
因此
φ – φ2 = π,
所以
φ = π + φ2 = 1.2π.
(3)如图所示.
4
4.14 三个同方向、同频率的简谐振动为
,
,
.
求:(1)合振动的圆频率、振幅、初相及振动表达式;
(2)合振动由初始位置运动到所需最短时间(A为合振动振幅).
[解答] 合振动的圆频率与各分振动的圆频率相同
ω = 314 = 100π(rad·s-1).
各分振动的振幅为A1 = A2 = A3 =0.08m,初相为φ1 =π/6、φ2 =π/2、φ3 =5π/6.
根据振动合成公式可得
Ax = A1cosφ1 + A2cosφ2 + A3cosφ3 = 0,
Ay = A1sinφ1 + A2sinφ2 + A3sinφ3
= 2A1 = 0.16(m),
合振幅为
= 0.16(m),
初位相为
φ = arctan(Ay/Ax) = π/2.
合振动的方程为
x = 0.16cos(100πt + π/2).
(2)当时,可得
,
解得
100πt + π/2 = π/4或7π/4.
由于t > 0,所以只能取第二个解,可得所需最短时间为t = 0.0125s.
4.15 将频率为384Hz的标准音叉振动和一待测频率的音叉振动合成,测得拍频为3.0Hz,在待测音叉的一端加上一小块物体,则拍频将减小,求待测音叉的固有频率.
[解答]标准音叉的频率为
v0 = 384(Hz),
拍频为
Δv = 3.0(Hz).
如果待测音叉的固有频率v2比标准音叉的频率大,则得
Δv = v2 - v0,
可能的频率是
v2 = v0 + Δv = 387(Hz).
如果待测音叉的固有频率v1比标准标准音叉的频率小,则得
Δv = v0 – v1,
可能的频率是
v1 = v0 - Δv = 381(Hz).
在待测音叉上加一小块物体时,相当于弹簧振子增加了质量,由于ω2 = k/m,可知其频率将减小.如果待测音叉的固有频率为v1,加一小块物体后,其频率v`1将更低,与标准音叉的拍频将增加;实际上拍频是减小的,所以待测音叉的固有频率v2,即387Hz.
4.16 示波器的电子束受到两个互相垂直的电场作用.电子在两个方向上的位移分别为x = Acosωt和y = Acos(ωt +φ).求在φ = 0,φ = 30o,及φ = 90o这三种情况下,电子在荧光屏上的轨迹方程.
[解答]根据公式
,
其中Δφ = φ2 – φ1 = -π/2,而φ1 = 0,φ2 = φ.
(1)当Δφ = φ = 0时,可得
,
质点运动的轨道方程为
y = x,
轨迹是一条直线.
(2)当Δφ = φ = 30o时,可得质点的轨道方程
,
即 ,
轨迹是倾斜的椭圆.
(3)当Δφ = φ = 90o时,可得
,
即 x2 + y2 = A2,
质点运动的轨迹为圆.
4.17 质量为0.4kg的质点同时参与互相垂直的两个振动:
,
.
式中x和y以米(m)计,t以秒(s)计.
(1)求运动的轨道方程;
(2)画出合成振动的轨迹;
(3)求质点在任一位置所受的力.
[解答](1)根据公式
,
其中位相差为
Δφ = φ2 – φ1 = -π/2,
所以质点运动的轨道方程为
.
(2)合振动的轨迹是椭圆.
(3)两个振动的圆频率是相同的ω = π/3,质点在x方向所受的力为

,
即 Fx = 0.035cos(πt/3 + π/6)(N).
在y方向所受的力为

,
即 Fy = 0.026cos(πt/3 - π/3)(N).
用矢量表示就是,其大小为
,
与x轴的夹角为
θ = arctan(Fy/Fx).
4.18 楼内空调用的鼓风机如果安装在楼板上,它工作时就会使整个楼产生讨厌的震动。为了减小这种震动,就把鼓风机安装在有4个弹簧支撑的底座上。鼓风机和底座的总质量为576kg,鼓风机的轴的转速为1800r/min(转每分)。经验指出,驱动频率为振动系统固有频率5倍时,可减震90%以上。偌按5倍计算,所用的每个弹簧的倔强系数应多大?
[解答] 驱动频率

由于4个弹簧并联,其等效劲度系数为每个弹簧的劲度系数的4倍,即k=4k1,
由
可得

第5章 波动
P210.
5.1 据报道,1976年唐山大地震时,当地某居民曾被猛地向上抛起高,设地震横波为简谐波,且频率为,波速为,它的波长多大?振幅多大?
[解答] 人离地的速度 及地壳上下振动的最大速度,为

地震波的振幅为

地震波的波长

5.2 已知一波的波动方程为
y = 5×10-2sin(10πt – 0.6x) (m).
(1)求波长、频率、波速及传播方向;
(2)说明x = 0时波动方程的意义,并作图表示.
[解答](1)与标准波动方程比较:
,
得 2π/λ = 0.6,
因此波长为
λ = 10.47(m);
圆频率为
ω = 10π,
频率为
v =ω/2π = 5(Hz);
波速为
u = λ/T = λv = 52.36(m·s-1).
传播方向沿着x轴正方向.
(2)当x = 0时波动方程就成为该处质点的振动方程
y = 5×10-2sin10πt = 5×10-2cos(10πt – π/2),
振动曲线如图.
5.3 已知波的波动方程为
y = Acosπ(4t – 2x)(SI).
(1)写出t = 4.2s时各波峰位置的坐标表示式,并计算此时离原点最近的波峰的位置,该波峰何时通过原点?
(2)画出t = 4.2s时的波形曲线.
[解答]波的波动方程可化为
y = Acos2π(2t – x),
与标准方程比较
,
可知:周期为T = 0.5s,波长λ = 1m.波速为u = λ/T = 2m·s-1.
(1)当t = 4.2s时的波形方程为
y = Acos(2πx – 16.8π)
= Acos(2πx – 0.8π).
令y = A,则
cos(2πx – 0.8π) = 1,
因此 2πx – 0.8π = 2kπ,(k = 0,±1,±2,…),
各波峰的位置为
x = k + 0.4,(k = 0,±1,±2,…).
当k = 0时的波峰离原点最近,最近为
x = 0.4(m).
通过原点时经过的时间为
Δt = Δx/u = (0 – x)/u = -0.2(s),
即:该波峰0.2s之前通过了原点.
(2)t = 0时刻的波形曲线如实线所示.经过t = 4s时,也就是经过8个周期,波形曲线是重合的;再经Δt = 0.2s,波形向右移动Δx = uΔt = 0.4m,因此t = 4.2s时的波形曲线如虚线所示.
[注意]各波峰的位置也可以由
cos(2πx – 16.8π) = 1
解得,结果为
x = k + 8.4,(k = 0,±1,±2,…),
取同一整数k值,波峰的位置不同.当k = -8时的波峰离原点最近,最近为x = 0.4m.
5.4 一平面波在介质中以速度u = 20m·s-1沿x轴负方向传播.已知在传播路径上的某点A的振动方程为y = 3cos4πt.
(1)如以A点为坐标原点,写出波动方程;
(2)如以距A点5m处的B点为坐标原点,写出波动方程;
(3)写出传播方向上B,C,D点的振动方程.
[解答](1)以A点为坐标原点,波动方程为
.
(2)以B点为坐标原点,波动方程为

.
(3)以A点为坐标原点,则xB = -5m、xC = -13m、xD = 9m,各点的振动方程为
,
,
.
[注意]以B点为坐标原点,求出各点坐标,也能求出各点的振动方程.
5.5 一列简谐波沿x轴正向传播,在t1 = 0s,t2 = 0.25s时刻的波形如图所示.试求:
(1)P点的振动表达式;
(2)波动方程;
(3)画出O点的振动曲线.
[解答](1)设P点的振动方程为
yP = Acos(ωt + φ),
其中A = 0.2m.
在Δt = 0.25s内,波向右传播了
Δx = 0.45/3 = 0.15(m),
所以波速为
u = Δx/Δt = 0.6(m·s-1).
波长为
λ = 4Δx = 0.6(m),
周期为
T = λ/u = 1(s),
圆频率为
ω = 2π/T = 2π.
当t = 0时,yP = 0,因此
cosφ = 0;
由于波沿x轴正向传播,所以P点在此时向上运动,速度大于零,所以
φ = -π/2.
P点的振动表达式为
yP = 0.2cos(2πt - π/2).
(2)P点的位置是xP = 0.3m,所以波动方程为

.
(3)在x = 0处的振动方程为
y0 = 0.2cos(2πt + π/2),
曲线如图所示.
5.6 一平面简谐波沿X轴正向传播,其振幅的圆频率分别为A和ω,波速为U,设t=0时的波形曲线如图18所示。
⑴写出此波的波动方程。(y=Acon[(ωt –ωx/u)+x/2])
⑵求距0点分别为λ/8,和3λ/8两处质点的振动方程。
⑶求距0点分别为λ/8,和3λ/8两处质点在t=0时的振动速度。
解:(1) 以O点为坐标原点.由图可知,该点振动初始条件为
,
所以 
波的表达式为 
(2)  处振动方程为

 的振动方程为

(3) 
t = 0,处质点振动速度

t = 0,处质点振动速度

5.7 一平面简谐波沿X轴正向传播,其振幅A=10cm,波的圆频率ω=7πrad·s-1,当t=1.0s时,x=10cm处的a质点正通过其平衡位置向Y轴负方向运动,而x=20cm处的B质点正通过Y=5.0cm点向Y轴正方向运动。设该波的波长λ>10cm,求该平面波的表达式。
解:设平面简谐波的波长为?,坐标原点处质点振动初相为?,则该列平面简谐波的表达式可写成  (SI)
t = 1 s时 
因此时a质点向y轴负方向运动,故
 ①
而此时,b质点正通过y = 0.05 m处向y轴正方向运动,应有

且  ②
由①、②两式联立得 = 0.24 m

∴ 该平面简谐波的表达式为
 (SI)
或  (SI)
5.8 一简谐波沿x轴正向传播,波长λ = 4m,周期T = 4s,已知x = 0处的质点的振动曲线如图所示.
(1)写出时x = 0处质点的振动方程;
(2)写出波的表达式;
(3)画出t = 1s时刻的波形曲线.
[解答]波速为u = λ/T = 1(m·s-1).
(1)设x = 0处的质点的振动方程为
y = Acos(ωt + φ),
其中A = 1m,ω = 2π/T = π/2.
当t = 0时,y = 0.5,因此
cosφ = 0.5,
φ = ±π/3.
在0时刻的曲线上作一切线,可知该时刻的速度小于零,因此
φ = π/3.
振动方程为
y = cos(πt/2 + π/3).
(2)波的表达式为

.
(3)t = 1s时刻的波形方程为
,
波形曲线如图所示.
5.9一弹性波在媒质中传播的速度u = 1×103m·s-1,振幅A = 1.0×10-4m,频率ν= 103Hz.若该媒质的密度为800kg·m-3,求:
(1)该波的平均能流密度;
(2)1分钟内垂直通过面积S = 4×10-4m2的总能量.
[解答](1)质点的圆频率为
ω = 2πv = 6.283×103(rad·s-1),
波的平均能量密度为
= 158(J·m-3),
平均能流密度为
= 1.58×105(W·m-2).
(2)1分钟内垂直通过面积S = 4×10-4m2的总能量为
E = ItS = 3.79×103(J).
5.10 在截面积为S的圆管中,有一列平面简谐波在传播,其波的表达式为 ,管中波的平均能量密度是w,则通过截面积S的平均能流是多少?
[解答]
通过截面积S的平均能流
I=wuS
由波的表达式得, 所求为
5.11一平面简谐声波在空气中传播,波速u = 340m·s-1,频率为500Hz.到达人耳时,振幅A = 1×10-4cm,试求人耳接收到声波的平均能量密度和声强?此时声强相当于多少分贝?已知空气密度ρ = 1.29kg·m-3.
[解答]质点的圆频率为
ω = 2πv = 3.142×103(rad·s-1),
声波的平均能量密度为
= 6.37×10-6(J·m-3),
平均能流密度为
= 2.16×10-3(W·m-2),
标准声强为
I0 = 1×10-12(W·m-2),
此声强的分贝数为
= 93.4(dB).
5.12 S1与S2为两相干波源,相距1/4个波长,S1比S2的位相超前π/2.问S1、S2连线上在S1外侧各点的合成波的振幅如何?在S2外侧各点的振幅如何?
[解答]如图所示,设S1在其左侧产生的波的波动方程为
,
那么S2在S1左侧产生的波的波动方程为

],
由于两波源在任意点x产生振动反相,所以合振幅为|A2 – A1|.
S1在S2右侧产生的波的波动方程为
,
那么S2在其右侧产生的波的波动方程为

,
由于两波源在任意点x产生振动同相,所以合振幅为A2 + A1.
5.13,如图所示,S1和S2为两相干波源,它们的振动方向均垂直于图面,发出波长为? 的简谐波,P点是两列波相遇区域中的一点,已知 ,,两列波在P点发生相消干涉.若S1的振动方程为 ,求S2的振动方程。
[解答].因为两列波在P点发生相消干涉.


5.14,如图所示,一平面简谐波沿x轴正方向传播,BC为波密媒质的反射面.波由P点反射, = 3? /4, =? 6.在t = 0时,O处质点的合振动是经过平衡位置向负方向运动.求D点处入射波与反射波的合振动方程.(设入射波和反射波的振幅皆为A,频率为?.)
解:选O点为坐标原点,设入射波表达式为
 则反射波的表达式是 
合成波表达式(驻波)为 
在t = 0时,x = 0处的质点y0 = 0,,
故得 
因此,D点处的合成振动方程是

5.15 两波在一很长的弦线上传播,设其表达式为:
,
,
用厘米、克、秒(cm,g,s)制单位,求:
(1)各波的频率,波长、波速;
(2)节点的位置;
(3)在哪些位置上,振幅最大?
[解答](1)两波可表示为
,
,
可知它们的周期都为
T = 0.5(s),
频率为
v = 1/T = 2(Hz);
波长为
λ = 200(cm);
波速为
u = λ/T = 400(cm·s-1).
(2)位相差
Δφ = πx/50,
当Δφ = (2k + 1)π时,可得节点的位置
x = 50(2k + 1)(cm),(k = 0,1,2,…).
(3)当Δφ = 2kπ时,可得波腹的位置
x = 100k(cm),(k = 0,1,2,…).
5.16 在固定端x = 0处反射的反射波表达式是,设反射波无能量损失,求入射波的表达式y1 以及形成的驻波的表达式y 。
[解答] 在固定端x = 0处反射,反射点是波节,入射波、反射波在该点的位相差为,反射波沿x轴正向传播,则入射波沿x轴负向传播其表达式为
y1=,
形成的驻波的表达式为
y=y1+y2=
=
5.17 A、B 为两个汽笛,其频率皆为500Hz,A 静止,B 以60m/s 的速率向右运动,在两个汽笛之间有一观察者O,以30m/s 的速度也向右运动.空气中的声速为330m/s,观察者听到的拍频为多少?
[解答] 观察者O接收到汽笛A发出的频率 为
观察者O接收到汽笛B发出的频率 为

则观察者听到的拍频为

5.18 海面上波浪的波长为120m,周期为10s。一只快艇以24m/s的速度迎浪开行,它橦击海浪的频率是多大?多长时间橦击一次?如果它顺浪开行,它橦击海浪的频率又是多大?多长时间橦击一次?
[解答] 波浪的速度

快艇迎浪开行,它橦击海浪的频率为

周期为

顺浪开行,它橦击海浪的频率

周期为

5.19.一声源的频率为1080Hz,相对地面以30m·s-1速率向右运动.在其右方有一反射面相对地面以65m·s-1的速率向左运动.设空气中声速为331m·s-1.求:
(1)声源在空气中发出的声音的波长;
(2)反射回的声音的频率和波长.
[解答](1)声音在声源垂直方向的波长为
λ0 = uT0 = u/ν0 = 331/1080 = 0.306(m);
在声源前方的波长为
λ1 = λ0 - usT0 = uT0 - usT0 = (u - us)/ν0
= (331-30)/1080 = 0.2787(m);
在声源后方的波长为
λ2 = λ0 + usT0 = uT0 + usT0 = (u + us)/ν0
= (331+30)/1080 = 0.3343(m).
(2)反射面接收到的频率为

= 1421(Hz).
将反射面作为波源,其频率为ν1,反射声音的频率为

= 1768(Hz).
反射声音的波长为

=0.1872(m).
或者 = 0.1872(m).
[注意]如果用下式计算波长
=0.2330(m),
结果就是错误的.当反射面不动时,作为波源发出的波长为u/ν1 = 0.2330m,而不是入射的波长λ1.

第6章 光的干涉
P238.
6.1.如图所示,在双缝干涉实验中,用波长为的光照射双缝和,通过空气后在屏幕E上形成干涉条纹.已知P点处为第三级明条纹,若将整个装置放于某种透明液体中,P点为第四级明条纹,求该液体的折射率。
[解答]
P点处为第三级明条纹,则和到P点的光程差为 。
若将整个装置放于某种透明液体中,P点为第四级明条纹,则和到P点的光程差为 。
n=4/3
6.2在空气中做杨氏双缝干涉实验,缝间距为d = 0.6mm,观察屏至双缝间距为D = 2.5m,今测得第3级明纹与零级明纹对双缝中心的张角为2.724×10-3rad,求入射光波长及相邻明纹间距.
[解答]根据双缝干涉公式sinθ = δ/d,其中sinθ≈θ,d = kλ = 3λ,可得波长为
λ = dsinθ/k = 5.448×10-4(mm) = 544.8(nm).
再用公式sinθ = λ/d = Δx/D,得相邻明纹的间距为
Δx = λD/d = 2.27(mm).
[注意]当θ是第一级明纹的张角时,结合干涉图形,用公式sinθ = λ/d = Δx/D很容易记忆和推导条纹间隔公式.
6.3 如图所示,平行单色光垂直照射到某薄膜上,经上下两表面反射的两束光发生干涉,设薄膜厚度为e,n1>n2,n2<n3,入射光在折射率为n1的媒质中波长为λ,试计算两反射光在上表面相遇时的位相差.
[解答]光在真空中的波长为
λ0 = n1λ.
由于n1 > n2,所以光从薄膜上表面反射时没有半波损失;由于n1 > n2,所以光从薄膜下表面反射时会产生半波损失,所以两束光的光程差为
δ = 2n2e + λ0/2,
位相差为
.
6.4用某透明介质盖在双缝干涉装置中的一条缝,此时,屏上零级明纹移至原来的第5条明纹处,若入射光波长为589.3nm,介质折射率n = 1.58,求此透明介质膜的厚度.
[解答]加上介质膜之后,就有附加的光程差
δ = (n – 1)e,
当δ = 5λ时,膜的厚度为
e = 5λ/(n – 1) = 5080(nm) = 5.08(μm).
6.5 折射率为1.50的两块标准平板玻璃间形成一个劈尖,用波长λ = 5004nm的单色光垂直入射,产生等厚干涉条纹.当劈尖内充满n = 1.40的液体时,相邻明纹间距比劈尖内是空气时的间距缩小Δl = 0.1mm,求劈尖角θ应是多少?
[解答]空气的折射率用n0表示,相邻明纹之间的空气的厚度差为
Δe0 = λ/2n0;
明纹之间的距离用ΔL0表示,则
Δe0 = θΔL0,
因此
λ/2n0 = θΔL0.
当劈尖内充满液体时,相邻明纹之间的液体的厚度差为
Δe = λ/2n;
明纹之间的距离用ΔL表示,则
Δe = θΔL,
因此
λ/2n = θΔL.
由题意得Δl = ΔL0 – ΔL,所以劈尖角为

= 7.14×10-4(rad).
6.6 为测量在硅表面的保护层SiO2的厚度,可将SiO2的表面磨成劈尖状,如图所示,现用波长λ = 644.0nm的镉灯垂直照射,一共观察到8根明纹,求SiO2的厚度.
[解答]由于SiO2的折射率比空气的大,比Si的小,所以半波损失抵消了,光程差为
δ = 2ne.
第一条明纹在劈尖的棱上,8根明纹只有7个间隔,所以光程差为
δ = 7λ,
SiO2的厚度为
e = 7λ/2n = 1503(nm) = 1.503(μm).
6.7 某平凹柱面镜和平面镜之间构成一空气隙,用单色光垂直照射,可得何种形状的的干涉条纹,条纹级次高低的大致分布如何?
[解答]这种情况可得平行的干涉条纹,两边条纹级次低,越往中间条纹级次越高,空气厚度增加越慢,条纹越来越稀.
6.8 设牛顿环实验中平凸透镜和平板玻璃间有一小间隙e0,充以折射率n为1.33的某种透明液体,设平凸透镜曲率半径为R,用波长为λ0的单色光垂直照射,求第k级明纹的半径.
[解答] 第k级明纹的半径用rk表示,则
rk2 = R2 – (R – e)2 = 2eR.
光程差为
δ = 2n(e + e0) + λ0/2
= kλ0,
解得
,
半径为
.
6.9当牛顿环装置中平凸透镜和平板玻璃间的空气充以某种透明液体时,某个亮条纹直径由1.40cm变为1.27cm,求液体的折射率
[解答] 当牛顿环装置中平凸透镜和平板玻璃间的空气充以某种透明液体时,某个亮条纹半径公式由变为

故 
6.10 在制作珠宝时,为了使人造水晶(n=1.5)具有强反射本领,就在其表面镀一层一氧化硅(n=2.0)。要使波长为560nm的光强烈反射,这镀层至少应多厚?
[解答] 由于在一氧化硅 –空气界面反射时有相位突变,所以反射光加强的条件是

k=1时,有最小厚度

6.11 光源发出波长可继续变化的单色光,垂直射入玻璃板的油膜上(油膜n = 1.30),观察到λ1 = 400nm和λ2 = 560nm的光在反射中消失,中间无其他波长的光消失,求油膜的厚度.
[解答]等倾干涉光程差为
δ = 2ndcosγ + δ`,
其中γ = 0,由于油膜的折射率比空气的大、比玻璃的小,所以附加光程差δ` = 0.
对于暗条纹,有
δ = (2k + 1)λ/2,
即 2nd = (2k1 + 1)λ1/2 = (2k2 + 1)λ2/2.
由于λ2 > λ1,所以k2 < k1,又因为两暗纹中间没有其他波长的光消失,因此
k2 = k1 – 1.
光程差方程化为两个
2nd/λ1 = k1 + 1/2,2nd/λ2 = k2 + 1/2,
左式减右式得
2nd/λ1 - 2nd/λ2 = 1,
解得
= 535.8(nm).
6.12白光垂直照射到空气中一厚度为3800的肥皂膜上,肥皂膜的折射率,在可见光的范围内(4000~7600),哪些波长的光在反射中增强?
[解答]若反射加强,则光程差应满足条件:

 
在可见光范围内,有
 
 
6.13 用迈克尔逊干涉仪可测量长度的微小变化,设入射光波长为534.9nm,等倾干涉条纹中心冒出了1204条条纹,求反射镜移动的微小距离.
[解答]反射镜移动的距离为
Δd = mλ/2 = 3.22×105nm = 0.322(mm).
6.14 在迈克尔逊干涉仪一支光路中,放入一折射率为n的透明膜片,今测得两束光光程差改变为一个波长λ,求介质膜的厚度.
[解答]因为δ = 2(n – 1)d = λ,所以
d = λ/2(n – 1).

第7章 光的衍射
P261.
7.1平行单色光垂直入射在缝宽为mm的单缝上。缝后有焦距为400mm的凸透镜,在其焦平面上放置观察屏幕。现测得屏幕上中央明条纹两侧的两个第三级暗纹之间的距离为8mm,则入射光的波长为?
[解答]
单缝衍射的暗条纹分布规律是
,(k` = 1,2,3,…),
测得屏幕上中央明条纹两侧的两个第三级暗纹之间的距离为8mm,
y3 – y-3` = 6fλ/a =8mm

7.2一单色平行光束垂直照射在宽度为1.0mm的单缝上,在缝后放一焦距为2.0 m的会聚透镜。已知位于透镜焦平面处的屏幕上的中央明条纹宽度为2.0 m,则入射光波长约为多少?
[解答]
单缝衍射的暗条纹分布规律是
,(k` = 1,2,3,…),
中央明纹的宽度为
Δy = y1 – y-1` = 2fλ/a =2.0mm

7.3 在某个单缝衍射实验中,光源发出的光含有两种波长λ1和λ2,并垂直入射于单缝上.假如λ1的第一级衍射极小与λ2的第三级衍射极小相重合,试问:
(1)这两种波长之间有什么关系;
(2)在这两种波长的光所形成的衍射图样中,是否还有其他极小相重合?
[解答](1)单缝衍射的暗条纹形成条件是
δ = asinθ = ±k`λ,(k` = 1,2,3,…),
k1` = 1和k2` = 3的条纹重合时,它们对应同一衍射角,由于因此
λ1 = 3λ2.
(2)当其他极小重合时,必有
k1`λ1 = k2`λ2,
所以 k2` = 3k1`,
当k1` = 2时k2` = 6,可见:还有其他极小重合.
7.4 单缝的宽度a = 0.40mm,以波长λ = 589nm的单色光垂直照射,设透镜的焦距f = 1.0m.求:
(1)第一暗纹距中心的距离;
(2)第二明纹的宽度;
(3)如单色光以入射角i = 30o斜射到单缝上,则上述结果有何变动?
[解答](1)单缝衍射的暗条纹分布规律是
,(k` = 1,2,3,…),
当k` = 1时,
y1 = fλ/a = 1.4725(mm).
(2)除中央明纹外,第二级明纹和其他明纹的宽度为
Δy = yk`+1 - yk` = fλ/a = 1.4725(mm).
(3)当入射光斜射时,光程差为
δ = asinθ – asinφ = ±k`λ,(k` = 1,2,3,…).
当k` = 1时,可得
sinθ1 = sinφ ± λ/a = 0.5015和0.4985,
cosθ1 = (1 – sin2θ1)1/2
= 0.8652和0.8669.
两条一级暗纹到中心的距离分别为
y1 = ftanθ1 = 579.6(mm)和575.1(mm).
当k` = 2时,可得
sinθ2 =asinφ ± 2λ/a = 0.5029和0.4971,
cosθ2 = (1 – sin2θ2)1/2
= 0.8642和0.8677.
两条二级暗纹距中心的距离分别为
y2 = ftanθ2 = 581.9(mm)和572.8(mm).
第二明纹的宽度都为
Δy = y2 – y1 = 2.3(mm),
比原来的条纹加宽了.
7.5 一单色平行光垂直入射于一单缝,若其第三级衍射明纹位置正好和波长为600 nm的单色光垂直入射该缝时的第二级衍射明纹位置一样,求该单色光的波长.
[解答]除了中央明纹之外,单缝衍射的条纹形成的条件是
,(k = 1,2,3,…).
当条纹重合时,它们对应同一衍射角,因此(2k1 + 1)λ1 = (2k2 + 1)λ2,
解得此单色光的波长为
= 428.6(nm).
.7.6 一双缝,缝距mm,两缝宽度都是mm,用波长为的平行光垂直照射双缝,在双缝后放一焦距m的透镜。求:
(1)在透镜焦平面处的屏上,双缝干涉条纹的间距;
(2)在单缝衍射中央亮纹范围内的双缝干涉亮纹数目。
解:双缝干涉条纹,
(1) 第k级亮纹条件,d sin=k?
第k级亮条纹位置:
xk = f tg≈f sin≈kf? / d
相邻两亮纹的间距:
x = xk+1-xk=(k+1)f? / d-kf? / d=f? / d
=2.4×10-3 m=2.4 mm
(2) 单缝衍射第一暗纹,a sin?1 =?
单缝衍射中央亮纹半宽度:
x0 = f tg?1≈f sin?1
≈f? / a=12 mm
x0?/x =5
∴ 双缝干涉第±5极主级大缺级,
∴ 在单缝衍射中央亮纹范围内,双缝干涉亮纹数目N = 9
分别为 k = 0,±1,±2,±3,±4级亮纹
或根据d / a = 5指出双缝干涉缺第±5级主大,同样得该结论.
7.7 一衍射光栅,每厘米有400条刻痕,刻痕宽为1.5×10-5m,光栅后放一焦距为1m的的凸透镜,现以λ = 500nm的单色光垂直照射光栅,求:
(1)透光缝宽为多少?透光缝的单缝衍射中央明纹宽度为多少?
(2)在该宽度内,有几条光栅衍射主极大明纹?
[解答](1)光栅常数为
a + b = 0.01/400 = 2.5×10-5(m),
由于刻痕宽为b = 1.5×10-5m,所以透光缝宽为
a = (a + b) – b = 1.0×10-5(m).
对于宽度为a的单缝来说,暗纹形成的条件是
asinθ = ±2k`(λ/2) = ±k`λ,
当θ很小时,θ = sinθ = tanθ = y/f,因此暗纹的位置是
,(k` = 1,2,3,…),
取k` = 1,得一级暗纹的位置
y±1 = fλ/a = 50(mm).
因此中央明纹的宽度为
Δy0 = 2y1 = 2fλ/a = 100(mm).
(2)对于光珊来说,形成明纹的必要条件是
(a + b)sinθ = kλ,
在衍射的中央明纹范围内,光珊衍射的最高级数为

=2.5,
取整数2,可知光珊衍射在单缝衍射中央有
k = 0,±1,±2
共5条主极大明纹.
另外,根据缺级条件
k/k` = (a + b)/a = 2.5 = 5/2,
可知:光栅干涉的第5级明纹出现在单缝衍射的第2级暗纹处,因而缺级;其他4根条纹近似等角度分布,各有两根在单缝衍射的中央明纹和一级明纹中.根据对称性,一级干涉明纹有两条,二级干涉明纹也有两条,包括中央明纹,共有5条干涉主极大明纹在单缝衍射的中央明纹宽度内.
7.8波长为600 nm的单色光垂直入射在一光栅上,第二、第三级主极大明纹分别出现在sinθ = 0.2及sinθ = 0.3处,第四级缺级,求:
(1)光栅常数;
(2)光栅上狭缝的宽度;
(3)屏上一共能观察到多少根主极大明纹
[解答](1)(2)根据缺级条件
(a + b)/a = k/k`,
由题意得k` = 1,k = 4.解得b = 3a.
再根据光栅方程
(a + b)sinθ = kλ,
可得狭缝的宽度为
a = kλ/4sinθ,
将k = 2,sinθ = 0.2或将k = 3,sinθ = 0.3代入上式,可得
a = 1500(nm).
刻痕的宽度为
b = 3a = 4500(nm),
光栅常数为
a + b = 6000(nm).
(3)在光栅方程中
(a + b)sinθ = kλ,
令sinθ =1,得
k =(a + b)/λ = 10.
由于θ = 90°的条纹是观察不到的,所以明条纹的最高级数为9.又由于缺了4和8级明条纹,所以在屏上能够观察到2×7+1 = 15条明纹.
7.9 以氢放电管发出的光垂直照射在某光栅上,在衍射角θ = 41o的方向上看到λ1 = 656.2nm和λ2 = 410.1nm的谱线重合,求光栅常数的最小值是多少?
[解答] 根据光栅方程得
(a + b)sinθ = k1λ1,
和 (a + b)sinθ= k2λ2,
因此
k2/k1 = λ1/λ2 = 1.6 = 16/10 = 8/5,
可见:k1最小取5,k2最小取8.因此最小光栅常数为
a + b = k1λ1/sinθ = 5000(nm).
此题与第四题的解法相同.
7.10两光谱线波长分别为和,其中,试证明:它们在同一级光栅光谱中的角距离,其中d是光栅常数,k是光谱级数。
证:据光栅方程有
 ①
 ②
∵  ②-①,得 
∴  

7.11 白光中包含了波长从400nm到760nm之间的所有可见光谱,用白光垂直照射一光栅,每一级衍射光谱是否仍只有一条谱线?第一级衍射光谱和第二级衍射光谱是否有重叠?第二级和第三级情况如何?
[解答] 由于白光是连续光谱,经过光栅衍射之后仍然是连续光谱,所以每一级衍射光谱都是一条光谱带,而不是一条光谱线.
根据光栅方程(a + b)sinθ = kλ,如果两种不同级的单色光重叠,它们的衍射角应该相同.
假设某波长可见光的2级衍射谱线与波长最长的可见光的一级谱线重叠,即
2λ = 1×760,
可得
λ = 380nm,
由于该波长比波长最短的可见光的波长400nm还要短,可知:第一级衍射光谱和第二级衍射光谱没有重叠.
假设某波长可见光的3级衍射谱线与波长最长的可见光的二级谱线重叠,即
3λ = 2×760,
可得
λ = 507nm,
该波长在可见光的波长之内,可知:第二级衍射光谱和第三级衍射光谱有重叠.
反过来,假设波长最短的可见光的3级衍射谱线与某波长的可见光的二级谱线重叠,即
3×400 = 2λ`,
可得
λ` = 600nm,
可知:第二级衍射光谱在600~760nm的波长范围与第三级衍射光谱在400~507nm波长范围有重叠.
7.12 一雷达的圆形发射天线的直径D=5 m,发射的无线电波的频率为v=300GHz,计算雷达发射的无线电波束的角宽度。
[解答] 雷达天线发射出去的无线电波,相当于通过天线圆孔后的衍射波,除衍射的中央主极大外,其他各级衍射次级大对接收都没有实际效果。 所以雷达发射的无线电波束的角宽度,就是圆孔衍射所形成的爱利斑的直径对应的角宽度,它等于爱利斑半径对应的角宽度 的2倍。故所求为
 =
7.13 迎面开来的汽车,其两车灯相距为1m,汽车离人多远时,两灯刚能为人眼所分辨?(假定人眼瞳孔直径d为3 mm,光在空气中的有效波长为λ = 500nm)
[解答]人眼的最小分辨角为
θ0 = 1.22λ/D = 2.033×10-4(rad),
当车很远时θ0 = w/l,所以距离为
l = w/θ0 = 4918(m).
7.14 在X射线衍射实验中,用波长从0.095nm到0.130nm连续的X射线以30o角入射到晶体表面.若晶体的晶格常数d = 0.275nm,则在反射方向上有哪些波长的X射线形成衍射极大?
[解答]30o是入射角,因此掠射角为
θ = 90o - 30o = 60o.
根据布喇格公式2dsinθ = kλ,得X射线形成衍射极大的波长为
λ = 2dsinθ/k,(k = 1,2,3,…).
级数k
1
2
3
4
5
6
波长λ(nm)
0.476
0.238
0.159
0.119
0.095
0.079
是否所求
No
No
No
Yes
Yes
No
数值和结果如表所示,
第8章 光的偏振
P287
8.1 两偏振片组装成起偏和检偏器,当两偏振片的偏振化方向夹角成30o时观察一普通光源,夹角成60o时观察另一普通光源,两次观察所得的光强相等,求两光源光强之比.
[解答]第一个普通光源的光强用I1表示,通过第一个偏振片之后,光强为
I0 = I1/2.
当偏振光通过第二个偏振片后,根据马吕斯定律,光强为
I = I0cos2θ1 = I1cos2θ1/2.
同理,对于第二个普通光源可得光强为
I = I2cos2θ2/2.
因此光源的光强之比
I2/I1 = cos2θ1/cos2θ2 = cos230o/cos260o = 3:1.
8.2 一束线偏振光和自然光的混合光,当它通过一偏振片后,发现随偏振片的取向不同,透射光的强度可变化四倍,求入射光束中两种光的强度各占入射光强度的百分之几?
[解答]设自然光强为I1,线偏振光强为I2,则总光强为
I0 = I1 + I2.
当光线通过偏振片时,线偏振光被吸收,最小光强为自然光强的一半,即
Imin = I1/2;
最大光强是线偏振光强与自然光强的一半之和,即
Imax = I2 + I1/2.
由题意得Imax/Imin = 4,因此
2I2/I1 + 1 = 4,
解得
I2 = 3I1/2.
此式代入总光强公式得
I0 = I1 + 3I1/2.
因此入射光中自然光强的比例为
I1/I0 = 2/5 = 40%.
由此可得线偏振光的光强的比例为
I2/I0 = 3/5 = 60%.
[讨论]如果Imax/Imin = n,根据上面的步骤可得
I1/I0 = 2/(n + 1),
I2/I0 = (n - 1)/(n + 1),
可见:n的值越大,入射光中自然光强的比例越小,线偏振光的光强的比例越大.
8.3 三个偏振片堆叠在一起,第一块与第三块偏振化方向互相垂直,第二块与第一块的偏振化方向互相平行,现令第二块偏振片以恒定的角速度ω0绕光传播方向旋转,如图所示.设入射自然光的光强为I0,试证明:此自然光通过这一系统后出射光强度为I = I0(1 – cos4ωt)/16.
[证明]自然光通过偏振片P1之后,形成偏振光,光强为
I1 = I0/2.
经过时间t,P3的偏振化方向转过的角度为
θ = ωt,
根据马吕斯定律,通过P3的光强为
I3 = I1cos2θ.
由于P1与P2的偏振化方向垂直,所以P2与P3的偏振化方向的夹角为
φ = π/2 – θ,
再根据马吕斯定律,通过P2的光强为
I = I3cos2φ = I3sin2θ
= I0(cos2θsin2θ)/2 = I0(sin22θ)/8
= I0(1 – cos4θ)/16,
即 I = I0(1 – cos4ωt)/16,证毕.
8.4 水的折射率为1.33,玻璃的折射率为1.50,当光由水射向玻璃时,起偏角为多少?若光由玻璃射向水时,起偏角又是多少?这两个角度数值上的关系如何?
[解答]当光由水射向玻璃时,水的折射率为n1,玻璃的折射率为n2,根据布儒斯特定律
tani0 = n2/n1 = 1.1278,
得起偏角为
i0 = 48.44o.
当光由玻璃射向水时,玻璃的折射率为n1,水的折射率为n2,根据布儒斯特定律
tani0 = n2/n1 = 0.8867,
得起偏角为
i0 = 41.56o.
可见:两个角度互为余角.
8.5 光在某两种介质的界面上的临界角是450,它在界面同一侧的起偏角是多少?
[解答] 临界角与折射率的关系为

在界面同一侧的起偏角是

=
8.6 如图所示,自然光以起偏角i0从空气射向水面,水中有一块玻璃板,若以玻璃反射之光亦为线偏振光,求水面和玻璃平面的夹角(n玻 = 1.50,n水 = 1.33).
[解答]根据布儒斯特定律:
tani0 = n水,
可得空气的起偏角为
i0 = arctann水 = 53.06°.
折射角为
γ = 90° - i0 = 36.94°.
再根据布儒斯特定律:
tani = n玻/n水 = 1.128,
可得玻璃的起偏角为
i = 48.44°.
水面和玻璃平面的夹角为
θ = i – γ = 11.5°.
[讨论]为了简便起见,设
n1 = n水,n2 = n玻,
那么
tani0 = n1,tani = n2/n1,
水面与玻璃平面的夹角为
θ = i – γ = (i + i0) - 90°,
因此


,
即 .
这是最终公式.代入数值得
tanθ = 0.2034,
夹角为
θ = 11.5°.
8.7 已知从一池静水的表面反射出来的太阳光是线偏振光,此时,太阳在地平线多大仰角处?
[解答]此时,太阳光射向水面的入射角为
此时,太阳在地平线的仰角为

8.10 一方解石晶体置于两平行的且偏振化方向相同的偏振片之间,晶体的主截面与偏振片的偏振化方向成30o,入射光在晶体的主截面内,求以下两种情况下的o光和e光强度之比.
(1)从晶体出射时;
(2)从检偏器出射时.
[解答](1)从偏振片入射到晶体的光分成o光和e光,o光垂直于主截面,e光平行于主截面.设入射偏振光的振幅为A,则
Ao = Asinθ,Ae = Acosθ,
当光从晶体出射时,o光和e光强度之比为
.
(2)从晶体出射的o光和e光入射到第二块偏振片时,只有沿偏振化方向的光能够通过,o光和e光的振幅为
A`o = A osinθ,A`e = A ecosθ,
当光从偏振片出射时,o光和e光强度之比为

.
8.12 某晶体对波长为632.8nm的光的主折射率为no = 1.66,ne = 1.49.用其制成适应于该波长光的1/4玻片,晶片至少要多厚?该波片的光轴方向如何?
[解答]对于1/4玻片,o光和e光的位相差为
,
当k = 0时晶片厚度最小
l = λ0/4(n o - ne) = 930.6(nm).
要形成圆偏振光,波片的光轴方向要与光的偏振化方向成45度角.